nbayar1212
Thanks Received: 22
Elle Woods
Elle Woods
 
Posts: 78
Joined: October 07th, 2012
 
 
 

Q11 - If the public library

by nbayar1212 Fri May 31, 2013 11:12 pm

There was no explanation for this question so I figured I'd give it a go.

The stimulus tells us that if the library was moved from Redville to Glenwood, the library would be within walking distance of a large number of library users because there are more people in Glenwood and its necessary for the library to be close to their homes if people are to decide to walk there.
We are looking for an AC that gives us reason to believe moving the library to Glenwood WOULD NOT place the library in an area that would be within walking distance of a large number of library users.

a) Tells us that more people who walk to the library live in Redville than Glenwood but this has no effect on the claim that once the library IS MOVED to Glenwood, the library would then be within walking distance of even more users.

b) is correct. It tells us that the population of library users is much smaller in central Glenwood than in central Redville which gives us a clear reason to doubt the conclusion. If this is true, maybe even if the library is placed within walking distance of more people in Glenwood, only a small fraction of them will be library users and not make up for the loss of library users that currently live in Redville.

c) This would strengthen the conclusion since it gives us reason to think more people will be using the library if its moved

d) Has no effect on the conclusion since our conclusion is concerned with placing the library within walking distance of more people

e) Has no effect for the same reason as D since it doesn't deal with nuance of the conclusion i.e. doesn't give us reason to doubt the library will now be placed within walking distance of more people.
User avatar
 
tommywallach
Thanks Received: 468
Atticus Finch
Atticus Finch
 
Posts: 1041
Joined: August 11th, 2009
 
 
 

Re: Q11 - If the public library

by tommywallach Wed Jun 05, 2013 1:04 pm

Hello Nbayar,

Great work here. However, there's one thing I'd take issue with.

Remember, when describing an argument, even in your head, always be very specific about breaking apart your conclusion and your premises. In this case, you mistake an important element of the conclusion.

"The stimulus tells us that if the library was moved from Redville to Glenwood, the library would be within walking distance of a large number of library users..."

This is your conclusion, but you only write "large number of library users," when the conclusion is clearly about a "larger number of library users." This is a critical distinction, and one that you mistake multiple times (i.e. a few lines down, you again write "We are looking for an AC that gives us reason to believe moving the library to Glenwood WOULD NOT place the library in an area that would be within walking distance of a large number of library users." This should be larger.)

It doesn't cause you to get the wrong answer, but it is a very critical part of the argument.

Otherwise, great work! Thanks for getting involved here on the forums!

-t
Tommy Wallach
Manhattan LSAT Instructor
twallach@manhattanprep.com
Image
 
nbayar1212
Thanks Received: 22
Elle Woods
Elle Woods
 
Posts: 78
Joined: October 07th, 2012
 
 
 

Re: Q11 - If the public library

by nbayar1212 Wed Jun 05, 2013 1:25 pm

I can see why that would be important. Thanks for clarification!
User avatar
 
WaltGrace1983
Thanks Received: 207
Atticus Finch
Atticus Finch
 
Posts: 837
Joined: March 30th, 2013
 
 
trophy
Most Thanked
trophy
Most Thankful
trophy
First Responder
 

Re: Q11 - If the public library

by WaltGrace1983 Mon Apr 28, 2014 1:34 pm

I took too long with (A) and (B) so I am going to write this out for my own benefit. Don't mind me if you don't want to! This is a weaken question.

    There are more people living in central Glenwood than in central Redville
    +
    People generally walk only if close to home
    →
    The library in central Glenwood would be within walking distance of a larger number of library users


The big thing to note here is that there is a jump from talking about number of people who live in a city to the number of people who will use the library. Just because there are more people living in Glenwood doesn't mean that it will be "within walking distance of a larger number of library users." There could be a rural town of 200 people vs. a big city of 200,000 people and you could still have more library users in the rural town. Just because there is more people doesn't mean more users.

Another thing to note is this idea of "walking distance." The answer choice doesn't touch on this but I feel like this is a shortcoming in the question. We know NOTHING about walking distance and it is a shame. The argument is additionally assuming that the library being in the same city means that it is "close to their homes." I can assure you that, if you live NYC, not everything in NYC is "close to your home." I may be wrong, but I feel like the wording of this question would be cleaned up on newer LSATs. Not explaining what "walking distance" is leaves a really bad taste in my mouth.

    (A) This hinges on the word "currently." However, what is currently true may not actually have anything to do with what will be true when the library is moved to Glenwood. Suppose more Redville residents do currently walk to the library than Glenwood residents. Does this say much? Not really. Maybe the library isn't currently within walking distance for the majority of people in Glenwood who would use the library - maybe it will be when the library is moved. This also doesn't say anything about the number of people in Glenwood who actually will use the library.

    (B) This also does use the word "currently" but it offers much more information that is helpful to destabilizing the argument that more library users will be within walking distance of the new library. This answer choice states that the number of people who would use the library in Glenwood is smaller than the number of people in Redville who currently use the library. Now I still don't exactly know anything about walking distance and whether or not the users from Redville will trek over to Glenwood but this does cast some doubt on the conclusion.

    (C) This might strengthen by showing that there will continually be more and more people using the library yin Glenwood. However, what about the walking distance idea?

    (D) We don't care about people that drive. We want people that walk!

    (E) So this is saying that most of the people who currently walk would still go to the library! That's great and it actually strengthens the argument by showing that there shouldn't be a big drop (if any) in the number of users and potentially will be even more users - as the argument predicts! However, we still don't know anything about walking distance.


Am I crazy to think that the lack of information on "walking distance" is a really big issue to this (and the previous) question on this preptest? I feel like, without this information about what is actually considered "walking distance," the answers are much more weak than they should be.